Family Midterm

Pataasin ang iyong marka sa homework at exams ngayon gamit ang Quizwiz!

The nurse has completed assessing the blood glucose levels of several infants who are 24 hours old. Which result should the nurse prioritize for intervention? 30 mg/dL 50 mg/dL 70 mg/dL 90 mg/dL

30 mg/dL

A nurse is providing education to a client who is 8 weeks' pregnant. The client stated she does not like milk. What is a source of calcium that the nurse can recommend to the client? dark, leafy green vegetables deep red or orange vegetables white bread and rice meat, poultry, and fish

dark, leafy green vegetables

The nursing instructor is reviewing with students ways to assess if the infant is getting enough at each feeding. Which of the following statements by a student demonstrates an understanding of these concepts? "Assess how long the infant breastfeeds at a time." "Assess the infant's voiding, growth, and alertness." "Assess how much formula is taken each time." "Assess how long the infant sleeps after each feeding."

"Assess the infant's voiding, growth, and alertness."

How can the nurse best counsel a patient with pyrosis? "It is important to continue to eat three meals a day." "Chest pain is common and is not a concern in pregnancy." "Avoid lying down 2 hours after eating." "Avoid sleeping in an upright position."

"Avoid lying down 2 hours after eating."

The nurse is assessng a newborn male in the presence of the parents and notes that he has a hypospadias. How should the nurse respond when questioned by the parents as to what this means? "He has normal male genitalia." "His testicles have not descended into the scrotal sac." "His urinary meatus in located on the under surface of the glans." "He has fluid in the scrotal sac."

"His urinary meatus in located on the under surface of the glans."

The nurse is conducting a teaching session for breastfeeding mothers. Which statement by a mother requires further clarification by the nurse? "I am glad I can have my two cups of coffee in the morning again." "I will continue to take a prenatal multivitamin as long as I am breastfeeding." "I will continue to add about 300 calories per day to my diet." "I will drink a large glass of water each time I nurse my baby."

"I am glad I can have my two cups of coffee in the morning again."

A nurse recommends to a client in labor to try concentrating intently on a photo of her family as a means of managing pain. The woman looks skeptical and asks, "How would that stop my pain?" Which explanation should the nurse give? "It distracts your brain from the sensations of pain." "It causes the release of endorphins." "It blocks the transmission of nerve messages of pain at the receptors." "It disrupts the nerve signal of pain via mechanical irritation of the nerves."

"It distracts your brain from the sensations of pain."

An 20-year-old primipara is getting ready to go home. She had a second-degree episiotomy with repair. She confides in the nurse that she is afraid to go to her postpartum checkup because she is afraid to have the stitches removed. Which reply by the nurse is best? "It doesn't hurt when the midwife takes out the stitches. You will only feel a little tugging and pulling sensation." "It is very important for you to go to your checkup visit. Besides, the stitches do not have to be removed." "Many women have that fear after having an episiotomy. The stitches do not need to be removed because the suture will be gradually absorbed." "Oh, you must not miss your follow-up appointment. Don't worry. Your midwife will be very gentle."

"Many women have that fear after having an episiotomy. The stitches do not need to be removed because the suture will be gradually absorbed."

The newborn weighing 6 lb 6 oz (2856 g), now weighs 5 lbs 14 oz (2632 g), 2 days later. Which response should the nurse prioritize to address the mother's concerns about the weight loss? "We need to do a more in-depth assessment." "This is a normal response." "How often are you feeding your baby?" "You may need to supplement breast-feedings for a while."

"This is a normal response."

A pregnant patient informs the nurse, "I am not going to do those leg exercises anymore. Every time I do them, I get terrible leg cramps." How can the nurse educate the patient to continue her exercise program and alleviate leg cramps? "When you are doing your leg exercises, point the toes to prevent leg cramps." "If you are having leg cramps, it is likely you are deficient in potassium and should take a supplement." "When you are doing your leg exercises, you can prevent leg cramps by extending the heel and never pointing the toes." "Stop all leg exercises. Leg cramps could be a sign of a blood clot in your legs."

"When you are doing your leg exercises, you can prevent leg cramps by extending the heel and never pointing the toes."

A client gave birth to a child 3 hours ago and noticed a triangular-shaped gap in the bones at the back of the head of her newborn. The attending nurse informs the client that it is the posterior fontanelle. The client is anxious to know when the posterior fontanelle will close. Which time span is the normal duration for the closure of the posterior fontanelle? 4 to 6 weeks 8 to 12 weeks 12 to 14 weeks 14 to 8 weeks

8 to 12 weeks

A client states that "she thinks" her water has broken. Which best provides confirmation of the rupture of membranes? Leakage from the perineum when the client coughs. Greenish fluid noted on the client's underwear A positive bacterial culture A positive Nitrazine test

A positive Nitrazine test

The nurse notices that there is no Vitamin K administration recorded on a newborn's medical record upon arrival to the newborn nursery. What would be the nurse's first action? Administer an oral dose of the Vitamin K to the newborn. Assume that the parents refused this medication for their infant. Call the Labor and Delivery nurse who cared for the newborn to inquire about why the medication was not documented. Give the IM dose of Vitamin K to prevent the possibility of hemorrhage in the newborn.

Call the Labor and Delivery nurse who cared for the newborn to inquire about why the medication was not documented.

The nurse is admitting a client who is in labor who reports her husband and doula will be arriving shortly. Which action should the nurse prioritize in response? Determine what activities the doula is qualified to handle Ask the client who she wants to come in first Continue with the admission assessment Print a copy of the instructions for the doula to sign off

Continue with the admission assessment

The nursing instructor is preparing a class discussing the role of the nurse during the labor and birthing process. Which intervention should the instructor point out has the greatest effect on relieving anxiety for the client? Massage therapy Continuous labor support Pharmacologic pain management Prenatal classes

Continuous labor support

A nursing student correctly identifies the childbirth preparation method that focuses on the premise that fear leads to tension, which leads to pain, is known as which of the following? psychosexual method Dick-Read method Lamaze method Bradley method

Dick-Read method

Which primary symptom does the nurse identify as a potentially fatal complication of epidural or intrathecal anesthesia? Difficulty breathing Staggering gait Decreased level of consciousness Intense pain

Difficulty breathing

The nurse is monitoring a client who is in the second stage of labor, at 2+ station, and anticipating birth within the hour. The client is now reporting the epidural has stopped working and is begging for something for pain. Which action should the nurse prioritize? Call the anesthetist from the nurse's station to retry the epidural. Call the primary care provider, and obtain a reduced dose of meperidine. Give the meperidine because she needs pain relief now. Encourage her through the contractions, explaining why she cannot receive any pain medication.

Encourage her through the contractions, explaining why she cannot receive any pain medication.

What term is used to describe the position of the fetal long axis in relation to the long axis of the mother? Fetal presentation Fetal attitude Fetal position Fetal lie

Fetal lie

All of the following are physical signs of protein deficiency in pregnancy EXCEPT Pale or brittle fingernails Dull, brittle, and lifeless hair Fissures at the corner of the mouth or pale mucous membranes Poor muscle tone or diminished reflexes

Fissures at the corner of the mouth or pale mucous membranes

A nurse is teaching a group of nursing students about the mechanism of labor when the fetus is in a cephalic presentation. The nurse determines the session is successful when the students correctly place the following events in which order? All options must be used. Extension External rotation Internal Rotation Flexion Expulsion

Flexion Internal rotation Extension External rotation Expulsion

The nursing instructor is conducting a teaching session illustrating the basics of feeding newborns. The instructor determines the class is successful after the students correctly choose which disorder as a contraindication to breastfeeding? Galactosemia Phenylketonuria Hypertension Hypothyroidism

Galactosemia

Which is the most important nursing assessment of the mother during the fourth stage of labor? The mother's psyche Blood pressure Hemorrhage Heart rate

Hemorrhage

The nurse is monitoring a client who is in labor and notes the client is happy, cheerful, and "ready to see the baby." The nurse interprets this to mean the client is in which stage or phase of labor? Transition phase Stage two Latent phase Stage three

Latent phase

Which nursing intervention offered in labor would probably be the most effective in applying the gate control theory for relief of labor pain? Encourage the woman to rest between contractions. Change the woman's position. Give the prescribed medication. Massage the woman's back.

Massage the woman's back.

A client has presented in the early phase of labor, experiencing abdominal pain and signs of growing anxiety about the pain. Which pain management technique should the nurse prioritize at this stage? Immersing the client in warm water in a pool or hot tub Practicing effleurage on the abdomen Administering a sedative such as secobarbital or pentobarbital Administering an opioid such as meperidine or fentanyl

Practicing effleurage on the abdomen

A nurse is providing care to a couple who is planning to get pregnant within a short time and who need proactive nutritional strategies prior to conceiving. The nurse would most likely recommend which type of care? Prenatal classes Lamaze classes A childbirth plan Preconception visits

Preconception visits

A nurse is coaching a woman during the second stage of labor. Which action should the nurse encourage the client to do at this time? Push with contractions and rest between them Hold the breath while pushing during contractions. Begin pushing as soon as the cervix has dilated to 8 cm. Pant while pushing.

Push with contractions and rest between them

A laboring client is restless and moving frequently in the bed. She appears to be more uncomfortable with the contractions but refuses pain medication when offered. The client's partner has left the room to stretch his legs. Which response by the nurse is most helpful? Stand silently at the back of the room. Stand next to the client at the side of the bed. Turn up the volume of music playing in the room. Turn on the television as a focal point.

Stand next to the client at the side of the bed.

A birth in which the lights are kept low, noise is kept to a minimum, and the newborn is placed into a warm bath immediately following delivery is called The Leboyer method The Dick-Read method Water birth Hydrotherapy

The Leboyer method

A nurse is caring for a female client in labor who has chosen hydrotherapy as her pain management for labor. As the nurse prepares the client for this treatment, which procedure is recommended as the most appropriate consideration? The client should be in active labor. The client, once submerged, should not stay in the tub too long. The water temperature should exceed the client's body temperature. The client should not submerge in water until membranes are ruptured.

The client should be in active labor.

A nurse is conducting an in-service program for staff nurses working in the labor and birth unit. The nurse is discussing ways to promote a positive birth outcome for the woman in labor. The nurse determines that additional teaching is necessary when the group identifies which measure? promoting the woman's feelings of control providing clear information about procedures allowing the woman time to be alone encouraging the woman to use relaxation techniques

allowing the woman time to be alone

The nurse is making a follow-up home visit to a woman who is 12 days postpartum. Which finding would the nurse expect when assessing the client's fundus? cannot be palpated 2 cm below the umbilicus 6 cm below the umbilicus 10 cm below the umbilicus

cannot be palpated

A client at 32 weeks' gestation receives an ultrasound that identifies intrauterine growth restriction. Which findings from the client's nutritional assessment would indicate to the nurse that additional teaching is needed? Select all that apply. eating large quantities of empty calorie foods difficulty eating because of continuing nausea history of gestational diabetes in previous pregnancy maternal age less than 18 years consuming 5 to 6 small meals each day

eating large quantities of empty calorie foods difficulty eating because of continuing nausea maternal age less than 18 years

A client in active labor is given spinal anesthesia. Which information would the nurse include when discussing with the client and family about the disadvantages of spinal anesthesia? passage of the drug to the fetus headache following anesthesia excessive contractions of the uterus increased frequency of micturition

headache following anesthesia

A nurse is observing the interaction between a new father and his newborn. The nurse determines that engrossment has yet to occur based on which behavior? demonstrates pleasure when touching or holding the newborn identifies imperfections in the newborn's appearance is able to distinguish his newborn from others in the nursery shows feelings of pride with the birth of the newborn

identifies imperfections in the newborn's appearance

A nurse is conducting an informal teaching session for a group of pregnant women about childbirth education classes. Which information would the nurse most likely include as an outcome associated of these types of classes? Select all that apply. Increased satisfaction Reduction in amount of reported pain Increased feelings of depression Increased feelings of control Reduction in complications

increased satisfaction reduction in amount of reported pain increased feelings of control

A woman calls the health care facility stating that she is in labor. The nurse would urge the client to come to the facility if the client reports which symptom? increased energy level with alternating strong and weak contractions moderately strong contractions every 4 minutes, lasting about 1 minute contractions noted in the front of abdomen that stop when she walks pink-tinged vaginal secretions and irregular contractions lasting about 30 seconds

moderately strong contractions every 4 minutes, lasting about 1 minute

Untreated hyperemesis can lead to preterm birth. What is the cause of the preterm birth? severe dehydration resulting in hypoperfusion of the placenta ketonuria resulting in neurologic changes in the fetus poor nutrient intake resulting in poor fetal growth class B drugs used to control the vomiting resulting in uterine contractions

severe dehydration resulting in hypoperfusion of the placenta

A mother of 4-year-old twin girls is expecting and asks the nurse if she can take her daughters to any classes to help prepare them for the birth of her new child. Which should the nurse recommend to this patient? expectant classes sibling classes preconception classes childbirth classes

sibling classes

The nurse is assessing a woman at 37 weeks' gestation who has presented with possible signs of labor. The nurse determines the membranes have ruptured based on which color of the Nitrazine paper? Pink Yellow Blue White

Blue

What type of common nonpharmacologic methods of pain relief can the nurse offer the client in labor? (Select all that apply.) Epidural anesthesia Guided imagery Effleurage Hydrotherapy Consciously controlled breathing

All but epidural anesthesia

A woman in scrubs enters a mother's room, while the nurse is completing an assessment. The woman states the doctor is in the nursery and has requested the infant be brought back for an examination. What will the nurse do? Ask the woman to bring the infant back when the doctor finishes the examination. Call the nursery to confirm the doctor does need this infant at this time. Ask the woman to see her hospital identification badge. Ask how long the infant will be gone since her next feeding is in 30 minutes.

Ask the woman to see her hospital identification badge.

The nurse is monitoring a client who is in active labor. The nurse will carefully monitor which phase of the involuntary uterine contraction to ensure the fetus is progressing adequately? Increment Acme Decrement Relaxation

Relaxation

When planning a labor experience for a primigravida, understanding which characteristic of labor pain is most helpful? All pain is the same. The characteristics of labor pain follow a pattern. Women innately know how to deal with labor pain. If the woman is in too much pain, a cesarean section is an option.

The characteristics of labor pain follow a pattern.

A nursing instructor is teaching about the importance of childbirth classes for pregnant women. Which of the following statements made by a student indicates a need for further instruction? "Childbirth classes give information only on how to prepare for childbirth." "Childbirth classes give information on how to prepare for childbirth and on how to parent." "Childbirth classes are informative for second-time mothers." "Childbirth classes are also available for siblings and grandparents."

"Childbirth classes give information only on how to prepare for childbirth."

A woman telephones her health care provider and reports that her water just broke. Which suggestion by the nurse would be most appropriate? "Call us back when you start having contractions." "Come to the clinic or emergency department for an evaluation." "Drink 3 to 4 glasses of water and lie down." "Come in as soon as you feel the urge to push."

"Come to the clinic or emergency department for an evaluation."

A young mother-to-be tells the nurse that she wants to breastfeed but only for about the first month. She adds that she wants to bottle feed at the same time. What should the nurse tell this patient? "Consider taking a breastfeeding class to learn more about it." "That sounds like a good plan." "Whatever works best for you is what you should do." "You really cannot do that."

"Consider taking a breastfeeding class to learn more about it."

A nurse has just taught a client about the signs of true and false labor. Which client statement indicates an accurate understanding of this information? "False labor contractions are regular." "False labor contractions intensify with walking." "False labor contractions usually occur in the abdomen." "False labor contractions move from the back to the front of the abdomen."

"False labor contractions usually occur in the abdomen."

A client is scheduled for a cesarean section under spinal anesthesia. After instruction is given by the anesthesiologist, the nurse determines the client has understood the instructions when the client states: "I can continue sitting up after the spinal is given." "I may end up with a severe headache from the spinal anesthesia." "The anesthesia will numb both of my legs to a level above my breasts." "I will need to lie on my right side to reduce vena cava compression."

"I may end up with a severe headache from the spinal anesthesia."

A client with a prepregnant BMI of 26 is concerned about gaining weight during pregnancy. Which statement by the client indicates an appropriate goal for this pregnancy? "I need to consume at least 1,500 nutrient-dense calories each day." "I will eliminate carbohydrates from my diet to control my weight." "I will eat two large meals with high protein content each day." "I am eating for two now, so the baby will burn the extra calories."

"I need to consume at least 1,500 nutrient-dense calories each day."

The nurse is completing the teaching for a newly pregnant client with a BMI of 23. Which statement by the client indicates an understanding of weight gain during this pregnancy? "I need to gain 25 to 35 pounds (11 to 16 kg) during this pregnancy." "I need to gain 0.5 pounds (0.23 kg) per week during this pregnancy." "I need to gain less than 25 pounds (11 kg) during this pregnancy." "I need to gain 1 pound (0.45 kg) per week throughout this pregnancy."

"I need to gain 25 to 35 pounds (11 to 16 kg) during this pregnancy."

A woman has informed the nurse that she plans to have a freebirth. Which statement indicates that the woman understands what freebirth entails? "I plan on giving birth at home without anyone around except my husband and my children." "I will be unlimited in the positions I am able to assume while in labor at the birthing center." "I will not have to wear fetal monitors while in labor at the hospital." "I will give birth at home under the care of a lay midwife."

"I will give birth at home under the care of a lay midwife."

A 22-year-old female is 25 weeks pregnant. She explains that she is having difficulty getting an adequate amount of protein into her diet because she is a vegetarian. How can the nurse best counsel this patient? "Try to eat at least 5 servings of fish a week." "In addition to getting protein from eating dairy, eat complementary proteins such as beans and rice together, or beans and wheat together." "Eat more leafy greens such as spinach and romaine lettuce and more vegetable oils, almonds, and avocados." "For your baby's growth and development, you are going to have to start to eat poultry."

"In addition to getting protein from eating dairy, eat complementary proteins such as beans and rice together, or beans and wheat together."

A new mother is concerned because it is 24 hours after birth and her breasts have still not become engorged with breast milk. How should the nurse respond to this concern? "It takes about 3 days after birth for milk to begin forming." "I'm sorry to hear that. There are some excellent formulas on the market now, so you will still be able to provide for your infant's nutritional needs." "You may have developed mastitis. I'll ask the primary care provider to examine you." "You are experiencing lactational amenorrhea. It may be several weeks before your milk comes in."

"It takes about 3 days after birth for milk to begin forming."

A first-time mother learning to breastfeed her newborn is worried her baby is not getting enough milk. Which statement made by the nurse is most appropriate? "Don't worry, your newborn will let you know if she is hungry by crying loudly." "Look at the number of wet and diapers with stool plus how many times the baby is feeding in a 24 hour period." "You can pump your breast and feed from a bottle. That way you can see the amount your newborn is eating with each feeding." "All mothers worry about this, but you will be able to tell if your infant is dehydrated by palpating the fontanels."

"Look at the number of wet and diapers with stool plus how many times the baby is feeding in a 24 hour period."

A nurse is giving discharge education to a group of new parents before they are discharged home with their infants. What information will the nurse include in the teaching? "Change the newborn's diaper every four hours while awake." "Place the newborn on the back to sleep and stomach to play." "Newborns can sleep on a couch to allow constant visual monitoring." "You need to give your newborn a bath everyday."

"Place the newborn on the back to sleep and stomach to play."

A nurse is providing prenatal education about microorganisms to avoid during pregnancy. Which of the following statements from the nurse is correct? "Vaccinations can protect pregnant women from most perinatal infections." "Pregnant women should avoid contact with dogs." "Pregnant women should avoid unpasteurized dairy products and undercooked meats." "Pregnant women should receive prophylactic antibiotics to prevent infection during their first trimester."

"Pregnant women should avoid unpasteurized dairy products and undercooked meats."

A newly pregnant patient explains to the nurse that she is interested in midwifery care but would like to deliver in the hospital. What is the best response from the nurse? "If you choose a midwife, you will have to deliver at home or in a birthing center." "Midwives are not allowed to deliver in the hospital setting." "That's no problem because most midwives deliver in the hospital setting." "Because you are a high-risk patient, you cannot have a midwife deliver your child."

"That's no problem because most midwives deliver in the hospital setting."

The health care provider approves a labor plan which includes analgesia. The client questions how analgesia will help her pain during labor. Which answer is best? "The analgesia will limit your ability to be out of bed without assistance." "The analgesia will block pain sensation and limit your ability to push." "The analgesia will reduce the sensation of pain for a limited period of time." "The analgesia will allow for a pain-free birth experience."

"The analgesia will reduce the sensation of pain for a limited period of time."

The nurse is admitting a primigravida client who has just presented to the unit in early labor. Which response should the nurse prioritize to assist the client in remaining calm and cooperative during birth? "The baby is coming. Relax and everything will turn out fine." "Do you want me to call in your family?" "Even though the baby is coming, the health care provider will be here soon." "The baby is coming. I'll explain what's happening and guide you."

"The baby is coming. I'll explain what's happening and guide you."

An experienced nurse is mentoring a graduate nurse and critiquing the graduate's shift handoff. Which statement requires clarification? "The client is experiencing lower back pain and I gave a backrub." "I changed the client position from her back to her side." "The client reports a pain level of 8. She has a low pain tolerance." "I instructed the client to ring if she felt the need to move her bowels."

"The client reports a pain level of 8. She has a low pain tolerance."

A woman in labor has chosen to use hydrotherapy as a method of pain relief. Which statement by the woman would lead the nurse to suspect that the woman needs additional teaching? "The warmth and buoyancy of the water has a nice relaxing effect." "I can stay in the bath for as long as I feel comfortable." "My cervix should be dilated more than 5 cm before I try using this method." "The temperature of the water should be at least 105℉ (40.5℃)."

"The temperature of the water should be at least 105℉ (40.5℃)."

A pregnant woman at the clinic tells the nurse that her goal is to not take any medications during labor. What is the nurse's best response to this patient? "If you absolutely do not want any medications, we will not give you any." "The ultimate goal of childbirth is a healthy baby and healthy parents. Sometimes medication is necessary." "If you end up taking any medication, then you will not achieve your goal." "That will not be a problem if that is what you and your husband want."

"The ultimate goal of childbirth is a healthy baby and healthy parents. Sometimes medication is necessary."

A nurse is performing an assessment on a client in early labor who is discouraged about the seemingly slow progress of her labor. Which response should the nurse prioritize for this client after noting the effacement is progressing even though the cervix is still only 2 cm for the past 2 hours? "You are still 2 cm dilated, but the cervix is thinning out nicely." "There has been no further dilatation; effacement is progressing." "You haven't dilated any further, but hang in there; it will happen eventually." Don't mention anything to the client yet; wait for further dilatation to occur.

"You are still 2 cm dilated, but the cervix is thinning out nicely."

When a client is counseled about the advantages of epidural anesthesia, which statement made by the counselor would indicate the need for further teaching? "Epidural anesthesia is more effective than opioid analgesia in providing pain relief." "You can continuously receive epidural anesthesia until you have the baby, and even afterward if you need it." "If you end up having a cesarean, the epidural can be used for anesthesia during surgery." "You have no trouble walking around and using the bathroom after you receive the epidural."

"You have no trouble walking around and using the bathroom after you receive the epidural."

A woman who gave birth to a healthy newborn 2 months ago comes to the clinic and reports discomfort during sexual intercourse. Which suggestion by the nurse would be most appropriate? "It takes a while to get your body back to its normal function after having a baby." "You might try using a water-soluble lubricant to ease the discomfort." "This is entirely normal, and many women go through it. It just takes time." "Try doing Kegel exercises to get your pelvic muscles back in shape."

"You might try using a water-soluble lubricant to ease the discomfort."

A young mother is concerned for her baby and asks the nurse if her baby is okay. What is the best response if the nurse notes: RR 66, nostrils flaring, and grunting sounds during respiration? "Your baby is fine, just learning how to breath." "Let's put a blanket around the baby; the baby is cold." "Your baby is having a little trouble breathing. I'll let the RN know." "Your baby is too warm. Let's take the blanket off."

"Your baby is having a little trouble breathing. I'll let the RN know."

The client is being rushed into the labor and delivery unit. At which station would the nurse document the fetus immediately prior to birth? -5 0 +1 +4

+4

A fetus is assessed at 2 cm above the ischial spines. How would the nurse document the fetal station? +4 +2 0 -2

-2

The nurse assesses the client and tells her the baby is at +1 station. Which is the best response by the nurse when asked by the client what this means concerning the location of the baby? 1 cm below the ischial spine. 1 cm below the symphysis pubis. 1 cm above the ischial spine. 1 cm above the symphysis pubis.

1 cm below the ischial spine.

A woman entering her third trimester of pregnancy asks the nurse how much weight she should gain during this trimester. What is the nurse's best answer? 0.5 lb/week 1 lb/week 1.5 lb/week 2 lbs/week

1 lb/week

The recommended weight gain during the first trimester for a woman who was in the normal weight range prepregnancy is 2 to 4 pounds. After the first trimester, what is the recommended weekly weight gain for a woman who was considered normal weight prepregnancy? 0.5 pound 0.66 pound 1 pound 2 pounds

1 pound

A patient of normal weight and new to the obstetric clinic asks the nurse how much weight to gain during pregnancy. What is the best response the nurse can give? 5 lbs over the first 3 months; 5 lbs over the second three months; 10 lbs over the last three months 7 lbs over the first 3 months; 7 lbs over the second three months; 7 lbs over the last three months 10 lbs over the first 20 weeks; 1 lb per week after that 5 lbs over the first 20 weeks; 2 lbs per week after that

10 lbs over the first 20 weeks; 1 lb per week after that

A low-risk client is in the active phase of labor. The nurse evaluates the fetal monitor strip at 10:00 a.m. and notes the following: moderate variability, FHR in the 130s, occasional accelerations, and no decelerations. At what time should the nurse reevaluate the FHR? 10:05 a.m. 10:30 a.m. 11:15 a.m. 11:30 a.m.

10:30 a.m.

A pregnant client of normal weight is concerned about excessive weight gain during her pregnancy. She states, "I don't want to get fat!" The nurse should inform her that she can expect to gain how much weight during her pregnancy? 20 to 25 pounds 25 to 30 pounds 15 to 20 pounds 35 to 40 pounds

25 to 30 pounds

Mrs. M is a 32-year-old female whose prepregnancy weight is considered "normal." She is carrying a single fetus. What is the recommended amount of total weight gain? 15 to 20 pounds 20 to 25 pounds 25 to 35 pounds 35 to 40 pounds

25 to 35 pounds

A nurse is serving as a doula to a client who is now in labor at an alternative birthing center. The client has opted for a water birth, and the nurse is now drawing the water into a large tub. What temperature should the nurse keep the water at? 37°C 35°C 39°C 41°C

37°C

A breastfeeding client asks how much breast milk to put in a bottle for her 3-month-old baby. The nurse will recommend placing how much breast milk in each bottle? 2 to 3 oz (57 to 85 g) 4 oz (113 g) 5 to 6 oz (142 to 170 g) 7 to 8 oz (198 to 227 g)

5 to 6 oz (142 to 170 g)

You are caring for an infant with a birth weight of 8 lbs. 5 oz. What would be an acceptable discharge weight for this infant? 7 lbs. 3 oz. 7 lbs. 5 oz. 7 lbs. 12 oz. 7 lbs. 1 oz.

7 lbs. 12 oz.

A G3 P2 with no apparent risk factors presents to the labor-and-delivery suite in early labor. She refuses the fetal monitor, stating she delivered her second baby at home without a monitor and everything went well. What is the nurse's best response? A few minutes on the monitor will ensure the baby is doing well and then the baby can then be monitored intermittently. Explain that you will have to call the physician and get an order to leave the fetal monitor off. Insist that the fetal monitor be used due to a lack of staff to adequately monitor her using any other method. Tell her that it is her decision, but that she will be placing herself and her baby at grave risk.

A few minutes on the monitor will ensure the baby is doing well and then the baby can then be monitored intermittently.

A nurse is reviewing the medical records of several women who come to the prenatal clinic for care. The nurse plans to increase the number of teaching sessions on nutrition for the women at risk for poor nutritional status. Which woman would the nurse most likely identify as needing the additional sessions. Select all that apply. A woman who is 42-years of age A woman whose prepartum weight was less 85% of ideal weight. A woman who shows a weight gain of 2 or more pounds per week. A woman who has elevated hemoglobin and hematocrit levels. A woman who engages in exercise of 30 minutes per day.

A woman who is 42-years of age A woman whose prepartum weight was less 85% of ideal weight. A woman who shows a weight gain of 2 or more pounds per week.

You are assessing the one minute APGAR score of a newborn baby. On assessment, you note the following about your newborn patient: heart rate 101, cyanotic body and extremities, no response to stimulation, no flexion of extremities, weak respiratory effort. APGAR 2 APGAR 3 APGAR 4 APGAR 5

APGAR 3

You are assessing the one minute APGAR score of a newborn baby. On assessment, you note the following about your newborn patient: heart rate 101, cyanotic body and extremities, no response to stimulation, no flextion of extremities, weak respiratory effort. APGAR 2 APGAR 3 APGAR 4 APGAR 5

APGAR 3

The nurse is assessing a laboring client and notes: 5 cm dilated, 80% effaced, zero station, contractions every 2 to 3 minutes, lasting 50 seconds, becoming increasingly uncomfortable, and apprehensive but appropriate and focused on breathing and relaxation. The nurse determines which nursing diagnosis is most appropriate for this client? Risk for altered tissue perfusion related to breathing techniques Impaired gas exchange related to prolonged contractions Acute pain related to uterine contractions Ineffective individual coping related to fear and anxiety

Acute pain related to uterine contractions

The nurse notices that a client in labor who is receiving an epidural has suddenly become drowsy. When the nurse asks her how she is doing, the client complains of a metallic taste in her mouth and blurred vision. Her speech is slurred. The nurse recognizes this as a serious complication related to the epidural. Which intervention would the nurse implement as a priority in this situation? Select all that apply. Administer oxygen to the client Administer an anticonvulsant to the client Prepare for prompt birth of the fetus Administration of 500 mL of IV Ringer's lactate Administration of a narcotic Administration of aspirin

Administer oxygen to the client Administer an anticonvulsant to the client Prepare for prompt birth of the fetus

A client in labor has received a spinal epidural block. Which nursing intervention should the nurse prioritize after assessing maternal hypotension and changes in the fetal heart rate (FHR)? Change the client to the supine position. Administer supplemental oxygen. Lower the woman's legs Stop IV fluid administration.

Administer supplemental oxygen.

A client has opted to receive epidural anesthesia during labor. Which of the following interventions should the nurse implement to reduce the risk of a significant complication associated with this type of pain management? Administration of 500 mL of IV Ringer's lactate Administration of 1000 mL of IV glucose solution Move the woman into a supine position Administration of aspirin

Administration of 500 mL of IV Ringer's lactate

A client in labor has requested the administration of narcotics to reduce pain. At 2 cm cervical dilatation, she says that she is managing the pain well at this point but does not want it to get ahead of her. What should the nurse do? Advise the client to hold out a bit longer, if possible, before administration of the drug, to prevent slowing labor. Agree with the client, and administer the drug immediately to keep the pain manageable. Explain to the client that narcotics should only be administered an hour or less before birth. Refuse to administer narcotics because they can develop dependency in the client and the fetus.

Advise the client to hold out a bit longer, if possible, before administration of the drug, to prevent slowing labor.

A pregnant client is discussing the possibility of using the services of an alternative birthing center (ABC). She states to the nurse, "I will be able to have an epidural if I choose too, right?" What information should the nurse provide to the client? Although there are many benefits to having the birth in an ABC, epidurals are administered in the hospital setting. Epidurals will be provided at the request of the client in any setting. There will be no reason for the client to have an epidural in the ABC because personnel there will liberally administer analgesic medications. The client will not need an epidural because she will be using distraction techniques.

Although there are many benefits to having the birth in an ABC, epidurals are administered in the hospital setting.

Which of the following statements regarding weight and pregnancy is correct? An underweight woman should increase her caloric intake by 500 to 1000 calories a day. Obesity usually occurs from hypothyroidism. Dieting during pregnancy to reduce weight is recommended only for morbidly obese women. Women who are underweight coming into pregnancy should gain the same amount of weight as women with a normal BMI.

An underweight woman should increase her caloric intake by 500 to 1000 calories a day.

Which statement is true regarding analgesia versus anesthesia? Analgesia only reduces pain, but anesthesia partially or totally blocks all pain in a particular area. Decreased FHR variability is a common side effect when regional anesthesia is used. Regional anesthesia should be given with caution close to the time of birth because it crosses the placenta and can cause respiratory depression in the newborn. Hypotension is the most common side effect when systemic analgesia is used.

Analgesia only reduces pain, but anesthesia partially or totally blocks all pain in a particular area.

A nurse recommends to a couple, who are soon to be parents that they attend early prenatal classes. Which of the following topics would be covered in an early prenatal class? Anatomy and physiology of childbirth and choices Physical and emotional changes of pregnancy Supply list and information on nutrition to prevent anemia Desire for and confidence in vaginal birth and risks of cesarean

Anatomy and physiology of childbirth and choices

A primigravida client has just arrived in early labor and is showing signs of extreme anxiety over the birthing process. Why should the nurse prioritize helping the client relax? Anxiety will increase blood pressure, increasing risk with an epidural. Decreased anxiety will increase trust in the nurse. Anxiety can slow down labor and decrease oxygen to the fetus. Increased anxiety will increase the risk for needing anesthesia.

Anxiety can slow down labor and decrease oxygen to the fetus.

You are assessing the one minute APGAR score of a newborn baby. On assessment, you note the following about your newborn patient: heart rate 130, pink body with cyanotic hands and feet, weak cry, flexion of the arms and legs, active movement. Apgar 7 Apgar 8 Apgar 9 Apgar 10

Apgar 8

A client who has just given birth to a healthy newborn required an episiotomy. Which action would the nurse implement immediately after birth to decrease the client's pain from the procedure? Offer warm blankets. Encourage the woman to void. Apply an ice pack to the site. Offer a warm sitz bath.

Apply an ice pack to the site.

A nurse is attending a seminar on the psychology and physiology of the pain of labor. Which action would indicate that the nurse has understood the information and has integrated this knowledge in the plan of care for a woman in labor? Select all that apply. Asks the client for her preferences in managing pain Provides non-pharmacological measures Assists in measures to enhance client's endorphin release Administers rapid pharmacological treatment Provides suggestions for choices in analgesia for relief

Asks the client for her preferences in managing pain Provides non-pharmacological measures Assists in measures to enhance client's endorphin release

The nurse is reviewing the medication administration record (MAR) of a client at 39 weeks' gestation and notes that she is ordered an opioid for pain relief. Which is an assessment priority after administering? Assess maternal blood pressure. Assess for constipation. Assess for dry mouth. Assess fetal heart rate.

Assess fetal heart rate

The nurse is reviewing the medication administration record (MAR) of a client at 39 weeks' gestation and notes that she is ordered an opioid for pain relief. Which is an assessment priority after administering? Assess maternal blood pressure. Assess for constipation. Assess for dry mouth. Assess fetal heart rate.

Assess fetal heart rate.

The nurse has just applied a sterile pressure dressing to an epidural site after removing the epidural catheter in a client who is now recovering from a standard delivery. Which action should the nurse now prioritize? Assess return of sensory and motor functions to the lower extremities. Help the client get up and walk around immediately. Let the client rest and recover while keeping her legs slightly elevated. Make sure the client receives plenty of fluids.

Assess return of sensory and motor functions to the lower extremities.

A client has just received combined spinal epidural. Which nursing assessment should be performed first? Assess vital signs. Assess pain level using a pain scale. Assess for progress in labor. Assess for spontaneous rupture of membranes. Assess for fetal tachycardia.

Assess vital signs.

A client has just had an epidural placed. Before the procedure, her vital signs were as follows: BP 120/70, P90 bmp, R18 per min, and O2 sat 98%. Now, 3 minutes after the procedure, the client says she feels lightheaded and nauseous. Her vital signs are BP 80/40, P100 bmp, R20 per min, and O2 sat 96%. Which interventions should the nurse perform? Assist the client to semi-Fowler's position, assess the fetal heart rate, start an IV bolus of 500 mL, and administer oxygen via face mask. Assist the client to a sitting position, assess the fetal heart rate, give naloxone, and administer oxygen via face mask. Assist the client to the supine position, recheck the blood pressure, and administer an IV bolus of 1000 mL. Assist the client to Trendelenburg's, assess the fetal heart rate, and administer oxygen via face mask.

Assist the client to semi-Fowler's position, assess the fetal heart rate, start an IV bolus of 500 mL, and administer oxygen via face mask.

During which time is the nurse correct to document the end of the third stage of labor? Following fetal birth When pushing begins At the time of placental delivery When the mother is moved to the postpartum unit

At the time of placental delivery

The nurse is caring for an infant. Which nursing action will best facilitate psychosocial growth of the infant? Follow the nap and feeding schedule used at home. Be consistently attentive to the infant's basic needs. Allow the infant opportunities to self-soothe. Ensure the caregivers to bring blankets and toys from home.

Be consistently attentive to the infant's basic needs.

What birthing options will act with gravity and speed up the second stage of labor? Side-lying position Knee-chest position Supine position in the hospital bed Birthing chairs

Birthing chairs

The nurse is assessing a client who has just given birth and notes her prelabor vital signs reveal a temperature 98.8oF (37.1oC), blood pressure 120/70 mm Hg, HR 80, and RR 20. Which current vital sign assessment should the nurse prioritize? Shaking chills with a fever of 100.3° F (37.9° C) Blood pressure 90/50 mm Hg, pulse 120 beats/min, respirations 24 breaths/min. Bradycardia and excessive, soaking diaphoresis Blood loss of 250 mL and WBC 25,000 cells/mL

Blood pressure 90/50 mm Hg, pulse 120 beats/min, respirations 24 breaths/min.

A nurse is reading a journal article about childbirth and the role of doulas in childbirth. The nurse demonstrates understanding of the article when the nurse identifies which activity as a function of a doula? Caring for the mother's emotional needs Providing health promotion and illness prevention Educating the woman about breath awareness and relaxation Teaching partners how to do massage techniques

Caring for the mother's emotional needs

The nurse is monitoring a laboring client with continuous fetal monitoring and notes a decrease in FHR with variable deceleration to 75 bpm. Which intervention should the nurse prioritize? Administer oxygen. Increase her IV fluids. Change the position of the client. Notify the primary care provider.

Change the position of the client.

The nurse is preparing a birthing care plan for a pregnant client. Which factor should the nurse prioritize to achieve adequate pain relief during the birthing process? The client has the baby without any analgesic or anesthetic. The health care provider decides the best pain relief for the mother and family. Client priorities and preferences are incorporated into the plan. The nurse suggests alternative methods of pain relief.

Client priorities and preferences are incorporated into the plan.

A pregnant client informs the nurse that she is considering a "water birth" for the labor and delivery of her child. What does the nurse anticipate as potential complications for this type of birth? (Select all that apply.) Contamination of the bath water with feces as the mother pushes during labor Prolonged second stage of labor Uterine infection Premature rupture of membranes

Contamination of the bath water with feces as the mother pushes during labor Uterine infection

At 32 weeks' gestation a client with a BMI of 23 has gained 24 lb (11 kg). What is the nurse's recommendation for weight gain for the remainder of this pregnancy? Continue to gain approximately 1 lb (.45 kg) per week during this pregnancy. Watch the diet so no additional weight is gained during this pregnancy. Limit weight gain to less than 5 lb (2 kg) for the remainder of this pregnancy. Increase weight gain to 1.5 lb (0.68 kg) per week during this pregnancy.

Continue to gain approximately 1 lb (.45 kg) per week during this pregnancy.

A woman gave birth vaginally approximately 12 hours ago, and her temperature is now 100° F (37.8° C). Which action would be most appropriate? Continue to monitor the woman's temperature every 4 hours; this finding is normal. Notify the health care provider about this elevation; this finding reflects infection. Obtain a urine culture; the woman most likely has a urinary tract infection. Inspect the perineum for hematoma formation.

Continue to monitor the woman's temperature every 4 hours; this finding is normal.

After teaching a group of students about Grantley Dick-Read and his view of childbirth, which of the following if identified by the students as an activity advocated by Dick-Read indicates that the teaching was successful? Controlled breathing Physical exercises Abdominal stroking Pressure point application

Controlled breathing

The nurse walks into a client's room and notes a small fan blowing on the mother as she holds her infant. The nurse should explain this can result in the infant losing body heat based on which mechanism? Conduction Convection Radiation Evaporation

Convection

A woman in labor has received a pudendal block. The nurse would assess the woman for which condition? Postdural headache Decrease in maternal blood pressure Window of pain ins a small area of the abdomen Urinary retention due to bladder atony

Decrease in maternal blood pressure

It is most likely that the physician would consider performing an amnioinfusion if the EFM tracing shows which of the following? Consistent early accelerations, variability present, and occasional decelerations Flat line without variability and no decelerations Occasional mild variable decelerations and moderate variability present Deep variable decelerations more than 60 bpm below the baseline with every contraction

Deep variable decelerations more than 60 bpm below the baseline with every contraction

A nurse assesses a primigravida client in the eighth week of gestation. The client reports nausea and vomiting in the mornings. The client tells the nurse, "I'm not able to keep liquids down and I'm eating like a bird." The client also expresses concerns about hormonal changes and how the pregnancy will affect her physical appearance. Which client problem should the nurse assess first? Deficient fluid volume Disturbed body image Knowledge deficit Slow weight gain

Deficient fluid volume

The nurse notices while holding him upright that a 1-day-old newborn has a significantly indented anterior fontanelle. She immediately brings it to the attention of the physician. What does this finding most likely indicate? Dehydration Increased intracranial pressure Vernix caseosa Cyanosis

Dehydration

Which cardinal movement of delivery is the nurse correct to document by station? Descent Flexion Extension Internal rotation

Descent

A nurse is performing a physical assessment of a woman in labor. As part of her assessment, she examines the outer and inner surfaces of her lips. What is the best rationale for this assessment? Detection of herpes virus infection Detection of a respiratory infection Detection of anemia Detection of rales

Detection of herpes virus infection

The nurse is monitoring a client in the first stage of labor. The nurse determines the client's uterine contractions are effective and progressing well based on which finding? Engagement of fetus Dilation of cervix Rupture of amniotic membranes Bloody show

Dilation of cervix

A woman in her 20s explains to the nurse that she would like to eat more healthy foods during her pregnancy but complains about the high cost of food. She confides that she just makes minimum wage at her job and that things are tight for her financially. What would be the most appropriate intervention for the nurse to make? Discuss the WIC program with the client. Recommend that the client focus on purchasing and eating only fruits and vegetables, to save money. Offer some money to the client to help her with her groceries. Discuss the school lunch program with the client.

Discuss the WIC program with the client.

Which statement best describes a doula? Doulas provide continuous labor support but do not perform any clinical procedures. Doulas are birth coaches who perform minimal clinical procedures. If a woman desires to avoid an epidural during labor, she will need a doula. Doulas are midwives that do not practice midwifery.

Doulas provide continuous labor support but do not perform any clinical procedures.

The nurse is coaching the mother during labor and tells her to breathe slowly from the chest at a rate of 6 to 12 breaths per minute. For what type of contractions would this method of breathing be best? As the woman is pushing when fully dilated For prevention of pushing before full dilation Cervical dilation between 4 and 6 cm Early contractions

Early contractions

A client in the first trimester reports having nausea and vomiting, especially in the morning. Which instruction would be most appropriate to help prevent or reduce the client's compliant? Drink plenty of fluids at bedtime. Avoid foods such as cheese. Avoid eating spicy food. Eat dry crackers or toast before rising.

Eat dry crackers or toast before rising.

A client at 28 weeks' gestation is asking for a laxative for constipation. What action would the nurse recommend? Eat fiber rich foods. Take a fiber-based laxative. Use a water-based enema. Insert a glycerin suppository.

Eat fiber rich foods.

A pregnant woman and her partner are reviewing information about a local hospital as a possible birth setting. The couple determine that the hospital is a mother-friendly facility based on which information? Encourages use of constant fetal monitoring Demonstrates high rates for episiotomies Promotes the use of repeat cesarean births over VBACs Encourages a woman to assume the positions of her choice during labor and birth

Encourages a woman to assume the positions of her choice during labor and birth

The nurse is preparing a postpartum client for discharge 72 hours after birth. The client reports bilateral breast pain around the entire breast on assessment. The nurse predicts this is related to which cause after noting the skin is intact and normal coloration? Mastitis Blocked milk duct Engorgement Excessive oxytocin

Engorgement

A primigravida has been in labor for 18 hours and is finally moving into the second stage and is anxious to begin pushing. Which assessment should be prioritize at this time? Evaluate maternal vital signs Ensure cervix fully dilated Evaluate fetal heart monitor Ensure empty urinary bladder

Ensure cervix fully dilated

What are small unopened or plugged sebaceous glands that occur in a newborn's mouth and gums? Epstein's pearls milia stork bites Mongolian spots

Epstein's pearls

T/F: The risk of acquiring an infection during a birth stay is greater at an alternative birthing center than at a hospital.

False

At a routine prenatal visit, a client asks the nurse for information on the Bradley method of childbirth. Which of the following would the nurse identify as an active participant during childbirth using the Bradley method? Midwife Doula Father Nurse

Father

A woman relates to the nurse that she understands that dietary fat is bad for her and that she should avoid it during pregnancy. How should the nurse respond? Fats are essential during pregnancy, and vegetable oils are a good source. Fats are essential during pregnancy, and fish such as marlin and orange roughy are good sources. Fats are not essential during pregnancy and thus are optional. Fats should be avoided during pregnancy.

Fats are essential during pregnancy, and vegetable oils are a good source.

A nurse is caring for a breastfeeding client who reports engorgement. The nurse identifies that the client's condition is due to not fully emptying her breasts at each feeding. Which suggestion should the nurse make to help her prevent engorgement? Feed the baby at least every two or three hours. Apply cold compresses to the breasts. Provide the infant oral nystatin. Dry the nipples following feedings.

Feed the baby at least every two or three hours.

A client in labor stares at a silver locket that her partner holds in front of her during a contraction. What type of distraction technique is the client using? Focusing Guided imagery Effleurage Conscious relaxation

Focusing

A woman who is obese comes to the clinic because she thinks that she is pregnant. The pregnancy is confirmed. The nurse is developing a teaching plan for this woman about pregnancy and challenges associated with it related to obesity. The nurse would most likely plan to address which condition as being an increased risk? Gestational diabetes Placenta previa Large for gestational age (LGA) infant Difficult vaginal delivery Spina bifida

Gestational diabetes Large for gestational age infant Difficult vaginal delivery

A client in labor receiving epidural anesthesia develops hypotension. Which of the following would the nurse do first? Give an intravenous bolus of fluid Inject ephedrine immediately Call the anesthesiologist immediately Inject propranolol immediately

Give an intravenous bolus of fluid

A client in her second trimester of pregnancy complains to the nurse of heartburn. Which of the following should the nurse suggest to the client as a preventative measure? Drink plenty of fluids before bedtime Use extra pillows when sleeping Have small frequent meals Eat dry crackers and toast before rising

Have small frequent meals

A nurse is performing an assessment on a female client who gave birth 24 hours ago. On assessment, the nurse finds that the fundus is 2 cm above the umbilicus and boggy. Which intervention is a priority? Notify the primary care provider, and document the findings. Have the client void, and then massage the fundus until it is firm. Assess a full set of vital signs. Check and inspect the lochia, and document all findings.

Have the client void, and then massage the fundus until it is firm.

The nurse has received the results of a client's postpartum hemoglobin and hematocrit. Review of the client's history reveals a prepartum hemoglobin of 14 gm/dL and hematocrit of 42%. Which result should the nurse prioritize? Hemoglobin 13 gm/dL and hematocrit 40 percent in a woman who has given birth vaginally Hemoglobin 12 gm/dL and hematocrit 38 percent in a woman who has given birth vaginally Hemoglobin 11 gm/dL and hematocrit 34 percent in a woman who has given birth by cesarean Hemoglobin 9 gm/dL and hematocrit 32 percent in a woman who has given birth by cesarean

Hemoglobin 9 gm/dL and hematocrit 32 percent in a woman who has given birth by cesarean

The LPN assists the RN while performing the Ortolani maneuver on a newborn. When asked by the mother the reason for this maneuver, which is the best response from the nurse? Spinal column movement Shoulder movement Clavicles for dislocation Hip for dislocation

Hip for dislocation

A new mother is concerned about how she will know whether her newborn is receiving enough breast milk. The nurse should explain that which factor is the least accurate measure of how much nutrition the baby is receiving? How long the baby nurses Change in the baby's body weight Whether the baby is voiding Whether the baby is alert

How long the baby nurses

The nurse is assisting a primigravida woman make decisions and prepare for her baby. The nurse should point out that breast-feeding is not an option for this client based on which assessment finding? Breast implants Galactosemia Need to return to work quickly Human immunodeficiency virus infection

Human immunodeficiency virus infection

In providing culturally competent care to a laboring woman, which is a priority? Identify the decision maker within the family. Identify any cultural foods used prior to labor. Identify who is the support person during the labor. Identify how the client expresses labor pain.

Identify how the client expresses labor pain.

At which time does the nurse anticipate that the woman will need the most pain relief measures? In the latent phase of the first stage of labor At the beginning of the second stage of labor During the transition phase of the first stage of labor In the active phase of the first stage of labor

In the active phase of the first stage of labor

The nurse is assisting a client through labor, monitoring her closely now that she has received an epidural. Which finding should the nurse prioritize to the anesthesiologist? Dry, cracked lips Urinary retention Rapid progress of labor Inability to push

Inability to push

A nurse is conducting an informal teaching session for a group of pregnant women about childbirth education classes. Which information would the nurse most likely include as an outcome associated of these types of classes? Select all that apply. Increased satisfaction Reduction in amount of reported pain Increased feelings of depression Increased feelings of control Reduction in complications

Increased satisfaction Reduction in amount of reported pain Increased feelings of control

The nurse is preparing the nursing care plan for a newborn who was born via a cesarean delivery. Which diagnosis should the nurse prioritize? Ineffective thermoregulation related to heat loss to the environment Altered nutrition less than body requirement related to limited formula intake Altered urinary elimination related to postcircumcision status Ineffective airway clearance related to mucus and secretions

Ineffective airway clearance related to mucus and secretions

Which is true regarding mineral requirements in the newborn? Infants who are breastfed need supplemental iron. Tetany from inadequate calcium intake is likely to occur in a breastfed infant, not in a formula-fed infant. Mothers who are breastfeeding should drink spring water only. Infants who are formula-fed should drink an iron-enriched formula for at least 12 months.

Infants who are formula-fed should drink an iron-enriched formula for at least 12 months.

Pica is a craving for nonfood items such as laundry starch, clay, or ice. What problem is associated with pica? Excessive weight gain Iron deficiency Lead contamination Diarrhea

Iron deficiency

A nurse does an initial assessment on a newborn and notes a pulsation over the anterior fontanelle that corresponds with the newborn's heart rate. How would the nurse interpret this? This is an abnormal finding and needs to be reported immediately. If the fontanelle feels full, then this is normal. This finding is normal if the pulsation can also be palpated in the posterior fontanelle. It is normal to feel pulsations that correlate with the newborn's heart rate over the anterior fontanelle.

It is normal to feel pulsations that correlate with the newborn's heart rate over the anterior fontanelle.

A nurse observes a woman in labor and her spouse as they practice breathing exercises to manage the client's pain. Before each contraction, she takes a deep, cleansing breath and then breathes at a controlled pace. During each contraction, she gradually shifts from slow, deep breaths to shallow, rapid ones and then back to slow, deep ones. Her spouse coaches her and performs effleurage on her abdomen. The nurse recognizes that this couple is using which method of pain management? Lamaze Dick-Read Psychosexual Bradley

Lamaze

A client in her second trimester of pregnancy arrives at a health care facility reporting heartburn. What instructions should the nurse offer to help the client deal with heartburn? Select all that apply. Limit consumption of food before bedtime. Consume lots of liquids before bedtime. Sleep in a semi-Fowler's position. Avoid use of antacids. Avoid overeating.

Limit consumption of food before bedtime Sleep in a semi-Fowler's position Avoid overeating

Breast milk contains all of the following except: Immunoglobulins Lactoferrin Lysozyme Listeria

Listeria

The nurse is preparing a client in labor to receive a pudendal block with lidocaine. The nurse would place the client in which position for administration? Left lateral Prone Lithotomy Trendelenburg

Lithotomy

A nurse recommends to parents who are planning to become pregnant that they attend prenatal classes. Which of the following would be the parents' learning objectives for attending such a prenatal class? Promotion of full-term birth and adequate newborn birth weight Development of relaxation awareness and plan for support during labor Management of health conditions affecting pregnancy and birth Learning of the benefits of a trial labor and vaginal birth

Management of health conditions affecting pregnancy and birth

A client in labor has been given an epidural anesthetic. Which nursing assessment finding is most important immediately following the administration of epidural anesthesia? Maternal respirations decrease from 20 to 14 breaths/minute. Maternal blood pressure decreases from 130/70 to 98/50 mm Hg. Maternal pulse increases from 78 to 96 beats/minute. Maternal temperature increases from 99° F (37.2° C) to 100° F (37.8° C).

Maternal blood pressure decreases from 130/70 to 98/50 mm Hg.

During a physical assessment of a newborn, the nurse observes bluish markings across the newborn's lower back. The nurse interprets this finding as: milia. Mongolian spots. stork bites. birth trauma.

Mongolian spots.

A nurse counsels a pregnant woman regarding her recommended daily allowance of calories. She advises her to obtain her carbohydrate calories from complex carbohydrates rather than simple carbohydrates. What is the best rationale for this guidance? More consistent regulation of glucose and insulin Provision of a greater amount of calories per gram Faster digestion of complex than simple carbohydrates Greater fatty acid content

More consistent regulation of glucose and insulin

The nurse is conducting an assessment on a newborn and witnesses a startled response with the extension of the arms and legs. The nurse should document this as which response? Fencing Moro Tonic neck Rooting

Moro

General anesthesia is not used frequently in obstetrics because of the risks involved. There are physiologic changes that occur during pregnancy that make the risks of general anesthesia higher than it is in the general population. What is one of those risks? The client is more sensitive to preanesthetic medications. The client is less sensitive to inhalation anesthetics. Neonatal depression is possible. Fetal hypersensitivity to anesthetic is possible.

Neonatal depression is possible.

Women who are obese in pregnancy are at higher risk for developing all of the following EXCEPT Macrosomia Pregnancy-induced hypertension Gestational diabetes Neural tube defects Cesarean birth

Neural tube defects

A nurse suggests to a client planning to start a family that she and her partner should attend childbirth education classes. Which of the following are taught in childbirth classes? Select all that apply. Crèche facilities Nutritional needs in pregnancy Stress management Healthy relationships Day nursery facilities

Nutritional needs in pregnancy Stress management Healthy relationships

A nurse is teaching a pregnant client how to perform tailor sitting. She cautions the client to put one leg in front of the other and to not put one ankle on top of another to avoid which of the following potential complications? Occluding blood supply to the lower legs Causing premature rupture of the membranes Losing balance and falling Overstretching the perineal muscles

Occluding blood supply to the lower legs

The nurse is conducting a prenatal class on breastfeeding. The nurse determines the class is successful when the young parents correctly choose which time frame is recommended for breastfeeding the infant? 4 months 6 months One year Two years

One year

The nutritional needs of an adolescent pregnant patient are unique because A pregnant adolescent should consume more calories than a pregnant 30-year-old with the same BMI Owing to typical food choices, an adolescent is often lacking calcium, iron, and folic acid in the diet Although most adolescents do not snack during the day, they usually have no problem finding time to consume three meals Adolescents are very independent and believe it is not necessary to talk with anyone else regarding their diet

Owing to typical food choices, an adolescent is often lacking calcium, iron, and folic acid in the diet

A nurse is educating a group of nursing students about the etiology of labor. Which of the following should the nurse explain as the hormone produced by the posterior pituitary? Progesterone Oxytocin Estrogen Relaxin

Oxytocin

A client is in active labor. As one of the nursing diagnoses is "Risk for trauma to the woman or fetus related to intrapartum complications or a full bladder," what would be appropriate for the nurse to do in order to achieve the goal of "no complications due to a full bladder"? Limit fluid intake to 300 mL every hour Insist the client use a bedpan every 2 hours Palpate the area above the symphysis pubis every 2 hours. Do a sterile "in and out" catheterization every 3 hours

Palpate the area above the symphysis pubis every 2 hours.

A client expresses a concern to the nurse about maintaining sexual responsiveness and preventing incontinence after giving birth. Which of the following exercises should the nurse most strongly recommend for these purposes? Pelvic floor contractions Squatting Tailor sitting Prenatal yoga

Pelvic floor contractions

A client is complaining of lower backache during her pregnancy. Which of the following exercises would best help her alleviate this pain? Pelvic rocking Pelvic floor contractions Tailor sitting Squatting

Pelvic rocking

What is the first action taken by a nurse caring for a newborn with suspected hypoglycemia? Check the client's blood sugar by a venous blood draw. Feed the newborn some formula immediately. Start an IV to provide intravenous glucose. Perform a heel stick to obtain a blood sample for testing for glucose level.

Perform a heel stick to obtain a blood sample for testing for glucose level.

You are performing a routine assessment on a mother post-delivery. The uterus is soft and displaced to the right of the umbilicus. What is your next nursing action? Perform fundal massage and assist the patient to the bathroom. Continue to monitor the mother. This is a normal finding post-delivery. Notify the physician. Administer PRN dose of Pitocin as ordered.

Perform fundal massage and assist the patient to the bathroom.

A client in the third trimester comes to the clinic for a follow up visit complaining of a backache. Which activity would the nurse include when teaching the client how to relieve her backache? Using a soft mattress Taking hot water baths Using warm tub baths Performing pelvic rocking exercises

Performing pelvic rocking exercises

A client in the third trimester comes to the clinic for a follow up visit complaining of a backache. Which activity would the nurse include when teaching the client how to relieve her backache? Using a soft mattress Taking hot water baths Using warm tub baths Performing pelvic rocking exercises

Performing pelvic rocking exercises

The nurse is preparing to administer an intradermal water injection to a client who is in labor. Which action should the nurse prioritize? Repeat the injection every 15 to 20 minutes as necessary Prepare four 1 mL syringes of 0.05 to 0.1 mL sterile water using a 25 gauge needle. These injections are very effective in relieving abdominal pain caused by contractions. Prepare two injections of 1 mL to 5 mL normal saline using a 10 mL syringe with a 25 gauge needle.

Prepare four 1 mL syringes of 0.05 to 0.1 mL sterile water using a 25 gauge needle.

The nursing instructor is teaching a group of nursing students about the various responsibilities of the labor and delivery medical team. The instructor determines the session is successful when the students correctly choose which function as the primary role of the LPN/LVN members of the team? Provide direct independent care to the client Assist the providers in the delivery room Provide care under the supervision of an RN Observatory to assist the RN

Provide care under the supervision of an RN

A nurse is explaining to a group of new parents about the changes that occur in the neonate to sustain extrauterine life, describing the cardiac and respiratory systems as undergoing the most changes. Which information would the nurse integrate into the explanation to support this description? The cardiac murmur heard at birth disappears by 48 hours of age. Pulmonary vascular resistance (PVR) is decreased as lungs begin to function. Heart rate remains elevated after the first few moments of birth. Breath sounds will have rhonchi for at least the first day of life as fluid is absorbed.

Pulmonary vascular resistance (PVR) is decreased as lungs begin to function.

A 32-year-old woman presents to the labor and birth suite in active labor. She is multigravida, relaxed, and talking with her husband. When examined by the nurse, the fetus is found to be in a cephalic presentation. His occiput is facing toward the front and slightly to the right of the mother's pelvis, and he is exhibiting a flexed attitude. How does the nurse document the position of the fetus? LOA LOP ROA ROP

ROA

A pregnant woman comes to the emergency department stating she thinks she is in labor. Which assessment finding concerning the pain will the nurse interpret as confirmation that this client is in true labor? Radiates from the back to the front Slows when the woman changes position Occurs in an irregular pattern Lasts about 20 to 25 seconds

Radiates from the back to the front

During a teaching session with a pregnant woman, the woman asks the nurse, "I've heard all sorts of information about salt. What's really true?" Which information about sodium would the nurse incorporate into the response? A woman should reduce her sodium intake by half during pregnancy. Retaining fluid in pregnancy is important for maintaining a pressure gradient that enables nutrients to cross the placenta. Most diets, regardless of culture, ethnicity, or background, are high in sodium. Inadequate sodium intake may result in fluid retention and may lead to peripheral edema.

Retaining fluid in pregnancy is important for maintaining a pressure gradient that enables nutrients to cross the placenta.

The nurse is monitoring a client and notes: contractions every 2 to 3 minutes, duration 45 to 60 seconds, strong intensity, cervix 10 cm, 100% effaced, fetal head crowns when client pushes. The nurse determines the client is currently in which stage or phase of labor? Transition Second Third Active

Second

A patient is concerned about the mercury levels in fish and asks the nurse which fish are safe to eat. The best response is Mackerel, king fish, tilefish, and shrimp Fresh salmon, shark, and swordfish Tilefish, shark, and tuna Shrimp, canned tuna, pollack, and catfish

Shrimp, canned tuna, pollack, and catfish

A newborn's one minute APGAR score is 5. Which of the following nursing interventions will you provide to this newborn/ Routine post-delivery care Continue to monitor and reassess the APGAR score in 10 minutes Some assistance such as drying the infant, rubbing the back and feet, and providing supplemental oxygen, and reassess APGAR score Full resuscitation assistance is needed and reassess APGAR score

Some assistance such as drying the infant, rubbing the back and feet, and providing supplemental oxygen, and reassess APGAR score

A client and her husband have prepared for a natural birth; however, as the client progresses to 8 cm dilation, she can no longer endure the pain and begs the nurse for an epidural. What is the nurse's best response? Suggest a less extreme alternative such as a sedative. Support the client's decision and call the obstetrician. Gently remind the client of her goal of a natural birth and encourage and help her. Ask the husband to gently remind her of their goal of natural birth and to encourage and help her.

Support the client's decision and call the obstetrician.

A client with hyperemesis gravidarum is on a clear liquid diet. Which foods would be appropriate for the nurse to serve? Select all that apply: Milk and ice chips Decaffeinated coffee and scrambled eggs Tea and gelatin Ginger ale and apple juice Cranberry juice and chicken broth Oatmeal and egg substitutes

Tea and gelatin Ginger ale and apple juice Cranberry juice and chicken broth

When needed, total parenteral nutrition (TPN) is often administered at home to reduce health care costs. When teaching a patient how to safely administer TPN, the home care nurse emphasizes Keeping the solution cold during administration Testing only fasting blood glucose to decrease the chance of infection Testing blood glucose every 6 hours Leaving the insertion site open to air to facilitate observation

Testing blood glucose every 6 hours

Which of the following is true regarding storing breast milk? Breast milk should never be frozen. Breast milk must be refrigerated immediately after pumping. Breast milk can still be used if it sits out overnight. Thawed breast milk must be used within 24 hours.

Thawed breast milk must be used within 24 hours.

A nurse is explaining the Apgar scoring to new mother and her partner. What should the nurse point out about this scoring method? Select all that apply. It is done at 1 and 5 minutes after birth. The baby is considered vigorous if the 5-minute score is above 7. Each factor receives a score of 0 or 2. The Apgar score is used to guide newborn resuscitation. The Apgar score is an immediate assessment of newborn cardiopulmonary adaptation.

The Apgar score is an immediate assessment of newborn cardiopulmonary adaptation. It is done at 1 and 5 minutes after birth. The baby is considered vigorous if the 5-minute score is above 7.

A woman expresses to you a desire to attempt to birth without pain medications. She likes the idea of a labor coach, but she is apprehensive about having anyone other than her husband and the doctor in the room at the time of delivery. What method of childbirth education would best suit this woman? The Bradley method The Lamaze method The Dick-Read method The psychosexual method

The Bradley method

A couple is writing out the birth plan for their labor and delivery. They desire an atmosphere with a dark, warm room and soft music playing. They opt for delayed cord clamping and an immediate warm bath after the infant is born. What type of birth does the nurse understand the parents are requesting? The Leboyer method A freebirth The Bradley method The Lamaze method

The Leboyer method

A nurse is assessing her female client in the labor admission unit. The client has been having contractions every 5 minutes for the past 6 hours. Which finding would the nurse use to determine if the client is experiencing true labor? The cervix has changes of effacement and dilatation. The client has a history of giving birth to two infants. The contractions increase in duration and intensity. The client's membranes ruptured spontaneously.

The cervix has changes of effacement and dilatation.

The nurse is notifying the health care provider that a client at 32 weeks' gestation reports bleeding. How best would the nurse report the data? The client states that she is having heavy bleeding. When ambulating the client to the bathroom, a gush of red blood was noted. The client has saturated three sanitary napkins in the past 4 hours. The client has lost 100cc of blood from what I approximate on her clothing.

The client has saturated three sanitary napkins in the past 4 hours.

Which psychosocial state is anticipated when the client enters the active phase of labor? The client will become more quiet and introverted. The client will become angry and begin to scream. The client will become more talkative and excited about the birth. The client will become tired and want the process over.

The client will become more quiet and introverted.

The nurse is assisting a client in labor and delivery and notes the placenta is now delivered. Which documentation should the nurse prioritize? The client's vital signs The end of recovery The completion of the third stage of labor The transition phase

The completion of the third stage of labor

What does the nurse understand is the benefit of educating the pregnant mother about performing Kegel exercises during her pregnancy? The exercises can help reduce pain and promote perineal healing in the postpartum period. The exercises will help shorten the first stage of labor. The exercises will relieve the backache associated with the laboring process. The exercises can reduce the discomfort felt from the uterine contractions.

The exercises can help reduce pain and promote perineal healing in the postpartum period.

The nurse is caring for a client at 39 weeks' gestation and whose fetal station is noted as a 0 (zero). The nurse is correct to document which? The client is fully effaced. The fetus is floating high in the pelvis. The fetus is in the true pelvis and engaged. The fetus has descended down the birth canal.

The fetus is in the true pelvis and engaged.

A breast-feeding mother calls the clinic, asking how much water she should be giving her 2-month-old infant. What would the nurse recommend to this mother? The mother should give the infant 4 to 6 ounces of water daily. The infant does not need any water supplement if nursing well. The mother should begin water feedings when the infant turns 4 months old. She can begin substituting one breast-feeding session with water beginning now.

The infant does not need any water supplement if nursing well.

A mother delivers her newborn and has chosen to formula-feed her baby. She asks the nurse how to keep her breasts from making milk. How would the nurse respond to the mother's question? The mother will produce milk after delivery but by manually expressing the milk, she can reduce the discomfort. The doctor can give her a hormone shot to dry up her breasts. When she becomes engorged, it is recommended to not wear a bra to allow the breasts to be more comfortable. The mother needs to understand that she will produce some milk, but wearing a constrictive bra will help dry up the milk supply.

The mother needs to understand that she will produce some milk, but wearing a constrictive bra will help dry up the milk supply.

The nurse is caring for four clients within the labor and delivery unit. Which client does the nurse anticipate will be sent home? The primigravid who is effaced, having intense contractions but at irregular intervals and dilation is 6 cm The multiparous who just experienced lightening and is having contractions 7 minutes apart. The primigravid who has a thinning cervix and a dilation of 3 cm The multiparous who is effaced with dilation of 4 cm.

The primigravid who has a thinning cervix and a dilation of 3 cm

The nurse is preparing new parents and their infant for discharge by answering questions and presenting basic discharge instruction. Which explanation should the nurse provide when questioned about the infant's yellow hue? The tint is due to jaundice. Yellow is the normal color for some newborns. The infant needs to be in the sunlight to clear the skin. It's a mild reaction to the vitamin K injection.

The tint is due to jaundice.

The pain of labor is influenced by many factors. What is one of these factors? The woman is prepared for labor and birth. The woman has a high tolerance for pain. The woman has a high threshold for pain. The woman has lots of visitors during labor.

The woman is prepared for labor and birth.

The nurse instructs the client about skin massage and the gate control theory of pain. Which statement would be appropriate for the nurse to include for client understanding of the nonpharmacologic pain relief methods? The gating mechanism is located at the pain site. Pain perception is decreased if anxiety is present. The gating mechanism opens so all the stimuli pass through to the brain. These methods are a technique to prevent the painful stimuli from entering the brain.

These methods are a technique to prevent the painful stimuli from entering the brain.

A client has just given birth to a healthy baby boy, but the placenta has not yet delivered. What stage of labor does this scenario represent? First Second Third Fourth

Third

The client in labor at 3 cm dilation and 25% effaced is asking the nurse for analgesia. Which explanation should the nurse provide when explaining why it is too early to administer an analgesic? This would cause fetal depression in utero. This may prolong labor and increase complications. The effects would wear off before delivery. This can lead to maternal hypertension.

This may prolong labor and increase complications.

The nurse is caring for a client in the postpartum period. The client has difficulty in voiding and is catheterized. The nurse would monitor the client for which condition? urinary tract infection loss of pelvic muscle tone increased urine output stress incontinence

UTI

Which nursing action would the nurse anticipate doing more often for a cesarean section newborn than a vaginal birth newborn? Monitor the temperature Assess voiding Note number of stools Upper airway suctioning

Upper airway suctioning

During pregnancy a woman's cardiovascular system expands to care for the growing fetus. After birth, during the early postpartum period, the woman eliminates the additional fluid volume she has been carrying. What is one way she does this? Urinary elimination Elimination of solid wastes Being too tired to eat Breathing off fluid vapor

Urinary elimination

Which of the following is not a physiologic basis for painful contractions during labor? The cervix is stretched. Uterine and cervical blood vessels dilate, increasing blood flow and causing hypoxia in the muscle fibers. The presenting part of the fetus puts additional pressure on adjacent organs. The descent of the fetus increases perineal pressure and stretching.

Uterine and cervical blood vessels dilate, increasing blood flow and causing hypoxia in the muscle fibers.

When developing a plan of care for a postpartum client, the nurse would identify which of the following as an expected outcome? Difficulty providing care for the newborn Foul smelling lochia Vital signs within acceptable limits Evidence of urinary retention

Vital signs within acceptable limits

You are doing client education with an obese woman who is trying to get pregnant. She confides in you that she is on a diet and trying to lose weight. Which of the following would you teach her? Any diet that helps to lose weight should be continued during pregnancy. Weight reduction should not be undertaken during pregnancy. Pregnancy is a safe time to lose weight. Obesity has no impact on pregnancy.

Weight reduction should not be undertaken during pregnancy.

There has been much research done on pain and the perception of pain. What is the result of research done on levels of satisfaction with the control of labor pain? Women report higher levels of satisfaction when regional anesthetics are used to control pain. Women report higher levels of satisfaction when they felt they had a high degree of control over the pain experience. Women report higher levels of satisfaction when the primary care provider makes the decision on what type of pain control to use. Women report higher levels of satisfaction when different types of relaxation techniques are used to control pain.

Women report higher levels of satisfaction when they felt they had a high degree of control over the pain experience.

A nurse is assisting a client who is in the first stage of labor. Which of the following principles should the nurse keep in mind to help make this client's labor and birth as natural as possible? Women should be able to move about freely throughout labor The support person's access to the client should be limited to prevent the client from becoming overwhelmed Routine intravenous fluid should be implemented A woman should be allowed to assume a supine position

Women should be able to move about freely throughout labor

The nursing instructor has finished leading a general discussion with a group of nursing students exploring the labor and birthing process. The instructor determines the session is successful when the students correctly state which goal should be a priority? Provide any medication the client requests. Dictate the pain management during labor for the best outcome. Work with the labor client to plan pain management options. Monitor the client for active labor, and assess for pain as needed.

Work with the labor client to plan pain management options.

The postpartum nurse is assessing clients, and all have given birth within the past 24 hours. Which client assessment leads the nurse to suspect the woman is experiencing postpartal blues? an 18-year-old mother who is currently holding her baby and looking face-to-face at the baby without saying a word a 29-year-old mother who has lots of family visiting and offering to help her with meals and cleaning for the next few months a 30-year-old woman who is teary-eyed when asked how she and the baby are doing with breastfeeding a 38-year-old G1 P1 who is constantly holding the baby and touching the baby's hands and fingers

a 30-year-old woman who is teary-eyed when asked how she and the baby are doing with breastfeeding

The nurse is reviewing the uterine contraction pattern and identifies the peak intensity, documenting this as which phase of the contraction? acme increment decrement diastole

acme

The coach of a client in labor is holding the client's hand and appears to be intentionally applying pressure to the space between the first finger and thumb on the back of the hand. The nurse recognizes this as which form of therapy? acupressure acupuncture effleurage biofeedback

acupressure

A nurse is monitoring the vital signs of a client 24 hours after birth. She notes that the client's blood pressure is 100/60 mm Hg. Which postpartum complication should the nurse most suspect in this client, based on this finding? bleeding postpartal gestational hypertension infection diabetes

bleeding

A client with hyperemesis gravidarium is started on TPN. What parameter does the nurse need to assess at least twice a day? blood glucose hemoglobin and hematocrit blood ketones potassium level

blood glucose

The nurse is teaching about an iron supplement that client is going to take every day. The nurse teaches the client to take the iron supplement with which type of fluid? citrus juice ice water low-fat milk hot tea

citrus juice

The nurse places a warmed blanket on the scale when weighing a newborn to minimize heat loss via which mechanism? evaporation conduction convection radiation

conduction

A nursing instructor is teaching about nonpharmacological techniques used for pain relief in labor. What are some techniques that the nurse would include in teaching? (Select all that apply.) conscious relaxation consciously controlled breathing effleurage focusing controlled screaming between contractions imagery

conscious relaxation consciously controlled breathing focusing imagery effleurage

A nursing student learning about childbirth preparation in a maternity course correctly identifies which of the following to be a major determinant of the plans women make for childbirth? religion culture social profession

culture

A postpartum client has a fourth-degree perineal laceration. The nurse would expect which medication to be prescribed? ferrous sulfate methylergonovine docusate bromocriptine

docusate

The nurse is measuring a contraction from the beginning of the increment to the end of the decrement for the same contraction. The nurse would document this as which finding? duration intensity frequency peak

duration

When teaching a group of nursing students about the stages of labor, the nurse explains that softening, thinning, and shortening of the cervical canal occur during the first stage of labor. Which term is the nurse referring to in the explanation? crowning effacement dilatation molding

effacement

A nurse is assessing a client's nutritional intake during pregnancy. What is the best method for accomplishing this? enacting a 24-hour nutrition recall weighing the client calculating the client's BMI having the client describe her food cravings

enacting a 24-hour nutrition recall

Which intervention would be least effective in caring for a woman who is in the transition phase of labor? having the client breathe with contractions providing one-to-one support encouraging the woman to ambulate urging her to focus on one contraction at a time

encouraging the woman to ambulate

A client presents with a positive home pregnancy test. The client has a 7-month-old baby and a 2-year-old child with her. What is a priority assessment to be completed during this visit? evaluation of nutrient status ultrasound for gestational dating screening for gestational diabetes antibody screening

evaluation of nutrient status

The nurse is describing pregnancy danger signs to a pregnant woman who is in her first trimester. Which danger sign might occur at this point in her pregnancy? dyspnea lower abdominal pressure swelling of extremities excessive vomiting

excessive vomiting

T/F: General anesthesia is a preferred method of pain management in childbirth

false

A nurse is providing care to a pregnant client in labor. Assessment of a fetus identifies the buttocks as the presenting part, with the legs extended upward. The nurse identifies this as which type of breech presentation? frank full complete footling

frank

After a normal labor and birth, a client is discharged from the hospital 12 hours later. When the community health nurse makes a home visit 2 days later, which finding would alert the nurse to the need for further intervention? presence of lochia serosa frequent scant voidings fundus firm, below umbilicus milk filling in both breasts

frequent scant voidings

A G1 P1 new mother asks what position she should place the newborn in while feeding formula? Which response by the nurse is best? supine, propping the bottle on a small pillow holding infant with head slightly elevated placing the baby side-lying in the crib totally sitting upright next to mother's body

holding infant with head slightly elevated

A woman who is breastfeeding her newborn reports that her breasts seem quite full. Assessment reveals that her breasts are engorged. Which factor would the nurse identify as the most likely cause for this development? cracking of the nipple improper positioning of infant inadequate secretion of prolactin inability of infant to empty breasts

inability of infant to empty breasts

When caring for a client with lactose intolerance, the nurse would be aware of which potential problem during pregnancy? inadequate calcium for skeletal growth inadequate protein for muscle development inadequate iron for red blood cell production inadequate folate for neural tube closure

inadequate calcium for skeletal growth

A nurse is discussing the benefits of childbirth education courses to a client who recently learned that she is pregnant. Which of the following are generally accepted as benefits of such classes? (Select all that apply.) Increased satisfaction in the birth experience Decrease in the amount of pain reported in childbirth Decrease in risky behaviors, such as smoking, during pregnancy Increased feelings of control during childbirth Increased likelihood to breastfeed Decreased incidence of birth defects

increased satisfaction in the birth experience decrease in amount of pain reported in childbirth decrease in risky behaviors, such as smoking, during pregnancy increased feelings of control during childbirth increased likelihood to breastfeed

A client who requested "no drugs" in labor asks the nurse what other options are available for pain relief. The nurse reviews several options for nonpharmacologic pain relief, and the client thinks effleurage may help her manage the pain. This indicates that the nurse will: lead the client through a series of visualizations to aid in relaxation. instruct the client or her partner to perform light fingertip repetitive abdominal massage. instruct the client to perform controlled chest breathing with a slow inhale and a quick exhale. press down firmly with her index finger and forefinger on key trigger points on the client's ankle or wrist.

instruct the client or her partner to perform light fingertip repetitive abdominal massage.

Assessment of a pregnant client reveals that she is experiencing Braxton Hicks contractions. Which finding would support this assessment? irregular pattern cervical dilation occurring typically very strong increasing duration

irregular pattern

A pregnant client has just received general anesthesia for the birth of her neonate. The client aspirates. The nurse anticipates the primary care provider will administer which medication? naloxone diphenhydramine nalbuphine isoproterenol

isoproterenol

Early in labor, a pregnant client asks why contractions hurt so much. Which answer should the nurse provide? lack of oxygen to the muscle fibers of the uterus due to compression of blood vessels release of endorphins in response to contractions distraction of the brain cortex by other stimuli blocking of nerve transmission via mechanical irritation of nerve fibers

lack of oxygen to the muscle fibers of the uterus due to compression of blood vessels

Assessment of a woman in labor reveals cervical dilation of 3 cm, cervical effacement of 30%, and contractions occurring every 7 to 8 minutes, lasting about 40 seconds. The nurse determines that this client is in: latent phase of the first stage. active phase of the first stage. transition phase of the first stage. perineal phase of the second stage.

latent phase of the first stage.

Nausea and vomiting are common reports during pregnancy. What nutritional action can be used to lessen nausea and vomiting? drinking liquids with meals limiting intake of heavy, greasy foods increasing fluid intake limiting carbohydrate intake

limiting intake of heavy, greasy foods

The nurse is assessing a client in labor for pain and notes she is currently not doing well handling the increased pain. Which opioid can the nurse offer to the client to assist with pain control? meperidine thiopental hydroxyzine hydrochloride secobarbital

meperidine

A woman in labor who received an opioid for pain relief develops respiratory depression. The nurse would expect which agent to be administered? butorphanol fentanyl naloxone promethazine

naloxone

The nurse completes the initial assessment of a newborn. Which finding would lead the nurse to suspect that the newborn is experiencing difficulty with oxygenation? respiratory rate of 54 breaths/minute abdominal breathing nasal flaring acrocyanosis

nasal flaring

A nursing instructor teaching about growth and development informs the students that the period when growth is the most rapid is which of the following? neonatal toddler years preschool adolescence

neonatal

Assessment reveals that the fetus of a client in labor is in the vertex presentation. The nurse determines that which part is presenting? shoulders occiput brow buttocks

occiput

A pregnant woman is taught many different types of exercises during pregnancy to help relieve pain, prevent complications, and facilitate an easier delivery. Which of the following exercises is taught to relieve backache during pregnancy? squatting pelvic floor exercises pelvic rocking abdominal muscle contractions

pelvic rocking

A nurse is monitoring a female client with an epidural block. Which complication would be the most important for the nurse to monitor in the client? accidental intrathecal block respiratory depression postdural puncture headache a failed block

respiratory depression

All the options are signs of respiratory distress in the newborn except: grunting. nasal flaring. chest retractions. central cyanosis. respiratory rate >50 breaths/minute. coughing.

respiratory rate >50 breaths/minute.

Client teaching is conducted throughout a client's hospitalization and is reinforced before discharge. Which self-care items are to be reinforced before discharge? resumption of intercourse activity resumption of prepregnancy diet signs and symptoms of infection infant formula selection

resumption of intercourse activity signs and symptoms of infection

The nurse is assessing a client at a postpartum visit. Which hemodynamic change will the nurse expect the client to exhibit? rise in hematocrit transient tachycardia increase in circulatory blood volume increase in cardiac output

rise in hematocrit

The nurse is working triage in the emergency department. The nurse realizes the client is in true labor when she states that the she is experiencing: contractions. expelling mucous plug. ruptured membranes. fetal engagement.

ruptured membranes.

A pregnant client reports chewing on ice throughout the day. Which laboratory value would the nurse evaluate? serum iron level serum potassium level serum glucose Level serum sodium level

serum iron level

A pregnant client is to receive fentanyl IV for pain control during labor. The nurse would assess the client for which possible effect? hypertension slowing of labor tachypnea depression

slowing of labor

What is a nursing intervention that helps prevent the most frequent side effect from epidural anesthesia in a pregnant client? administrating IV ephedrine administrating IV naloxone maintaining the client in a supine position starting an IV and hanging IV fluids

starting an IV and hanging IV fluids

The nurse educates the vegetarian client about which nutritional need during pregnancy? taking a B12 supplement limiting the intake of fiber supplementing the diet with vitamins A and C avoiding high intake of dark green vegetables.

taking a B12 supplement

T/F: According to Jamie Dunnum, the nurses are the memory makers when it comes to fetal, neonatal, and infant death

true

A pregnant woman has decided that she wants to receive epidural analgesia for pain control during labor and birth. After teaching the woman about this type of pain control, the nurse determines that additional teaching is needed when the woman identifies which condition as being associated with epidural analgesia? hypotension headache urinary frequency itching

urinary frequency


Kaugnay na mga set ng pag-aaral

INDIABIX - TNP_Pinoybix Section 4-5

View Set

Investment Vehicle Characteristics

View Set

the executive department and the office of the governor of texas (chapter 4)

View Set

Ch. 49- Drugs Affecting Corticosteroid Levels [fludrocortiSONE]

View Set

Contract Musts, Express and Implied Contracts, Bilateral and Unilateral Contracts

View Set

*U.S. History* Chapter 20-21 Test

View Set

Chapter 4 reproduction PrepU 240

View Set